You are on page 1of 38

Genitourinary Objectives Objective #1 State the indications, significance of abnormal findings and complications of the following Imaging and

Laboratory studies. Urinalysis, serum BUN and creatinine, creatinine clearance and GFR, IVP, cystoscopy, US of bladder kidney and prostate, Urine culture and sensitivity, renal biopsy Urinalysis: (types) Random = looks for protein and blood (typically for screening) First morning specimen = looks for by products of stuff like antigens, protein, etc. 24 hour specimen = (pee into a jug for 24 hours and put in fridge to keep cool) looks for creatinine, used to look for muscle break down. Indications: Used in the diagnosis of: o Calculi o Urinary tract infection (UTI) o Malignancy o Systemic dz affecting the kidney Used to screen for: o Pregnancy Significance of abnormal findings and complications: Color and odor: o Cloudy urine = precipitated phosphate crystals in alkaline urine or pyuria o Normal urine = urinoid odor doesnt imply infection o Fruity/sweet urine = diabetic ketoacidosis o Ammoniacal odor = alkaline fermentation after prolonged bladder retention Specific gravity (hydration status/concentrating ability of kidneys): o Normal USG 1.003-1.030 o Value < 1.010 = relative hydration o Value > 1.020 = relative dehydration o Increased USG = glycosuria and SIADH o Decreased USG = diuretic use, diabetes insipidus (DI), adrenal insufficiency, aldosteronism, & impaired renal fxn. Urine pH (generally reflects the serum pH, except in pt with renal tubular acidosis): o Possible range 4.5-8.0 normally slightly acidic (5.5-6.5) o Inability to acidify urine to a pH of < 5.5 despite an overnight fast and administration of an acid load = HALLMARK of renal tubular acidosis (RTA) o Alkaline urine in pt with UTI suggest urea splitting organism which may be associated with magnesium-ammonium phosphate crystals and can form staghorn calculi (struvite stones) o Uric acid calculi are associated with acidic urine Hematuria: o proteinuria, erythrocyte casts, and dysmorphic RBCs = Glomeular hematuria o proteinuria without erythrocyte casts or dysmorphic RBCs = Renal (nonglomerular) hematuria o absence of proteinuria, dysmorphic RBCs, and erythrocyte casts = Urologic hematuria Proteinuria (3 types): o Glomerular = (MC type) albumin is the primary urine protein o Tubular = results when malfxning tubule cells no longer metabolize or reabsorb normally filtered protein. Therefore low molecular weight proteins predominate over albumin o Overflow = low-molecular weight proteins overwhelm the ability of the tubules to reabsorb filtered protein. Glycosuria: o When the filtered load of glucose exceeds the ability of the tubule to reabsorb it = glycosuria common causes Diabetes mellitus, cushings syndrome, liver and pancreatic dz, and Fanconis syndrome

Ketonuria: o MC associated with uncontrolled diabetes, also occurs during pregnancy, carbohydrate-free diets, and starvation Nitrites: o Result of bacteria converting nitrates to nitrites, mostly gram negative but some gram positive organisms o Test is specific but not sensitive therefore doesnt rule out a UTI Leukocyte esterase (product of neutrophils): o If pyuria present usually associated with UTI o Causes of pyuria with negative cultures are Chlamydia, Ureaplasma ureanylticum, balanitis, urethritis, tuberculosis, bladder tumors, viral infxns, nephrolithiasis, foreign bodies, exercise, glomerulonephritis, and corticosteroid and cyclophosphamide. Bilirubin and urobilinogen: o Bilirubin in urine = Indicates further evaluation for liver dysfxn and biliary obstruction o Urobilinogen = hemolysis and hepatocellular dz elevate urobilinogen, abx and bile duct obstruction decrease urobilinogen in urine. Microscopic urinalysis: o Used to identify cells, casts, crystals, and bacteria Cells: Squamos epithelial cells indicate contamination Transitional epithelial cells is normal Renal tubule cells indicates significant renal pathology Dysmorphic RBC suggest glomerular dz Casts: Coagulum of Tamm-Horsfall mucoprotein and the trapped contents of tubule lumen indicate DCT or collecting duct issues dealing with urinary concentration or stasis, or when urinary pH is very low Types of casts = hyaline, erythrocyte, leukocyte, epithelial, granular, waxy, fatty, broad Crystals: Calcium oxalate = refractile square envelope Uric acid = yellow to orange-brown diamond or barrel shaped Triple phosphate = associated with alkaline urine and UTIs or may be normal coffin lid Cystine = colorless, hexagonal shape, present in acidic urine, which is diagnostic of cystinuira Bacteruria: Colony count of as low as CFU per mL suggest UTI Presence of bacteria in a male specimen is suggestive of infection and a culture should be obtained.

CMDT says Certain patterns are associated with renal dz o Bland urine sediment = chronic kidney dz and prerenal and postrenal disorders o Heavy proteinuria and lipiduria = nephrotic syndrome o Hematuria with dysmorphic RBCs, RBC casts, and proteinuria = glomerulonrphritis o Pigmented granular casts and renal tubular epithelial casts = acute tubular necrosis o WBCs, neutrophils, eosinophils, WBC casts, RBC, and small amounts of protein = interstitial nephritis and pyelonephritis o Pyruia alone = UTI ***Sorry I didnt summarize the 17 page monster NNB gave us**** Serum BUN and Creatinine Indications: Urea is an index used to measure renal fxn Synthesized mainly by the liver and is the end product of catabolism Urolithiasis/ureteral stones Renal cystic dz

Drug induce nephropathy Chronic renal dz

Significance of abnormal findings and complications: Conditions that cause increased BUN: dehydration, reduced effective circulating blood volume (prerenal azotemia), catabolic states (gastrointestinal bleeding, corticosteroid use), high protein diets, tetracycline, Conditions that decrease BUN: liver dz, malnutrition, sickle cell anemia, SIADH Complications of renal cystic dz = pain, hematuira, renal infections, renal stones, HTN, cerecral aneurysms Complications of chronic renal dz = HTn or hypOten, castrointestinal hemorrhage, Adult respiratory distress syndrome Creatinine Clearance/GFR Indicatioins Useful index of overall renal fxn Adults with proteinuria > grams per 24 hrs require this measurement Renal cystic dz Drug induce nephropathy Chronic renal dz Significance of abnormal findings and complications: Creatinine clearance is one way to estimate GFR, creatinine is a product of muscle metabolism produced at a normal rate and cleared by renal excretion, and excretion and production are normally equal. Conditions that elevate creatinine: ketoacidosis, cephalothin, cefoxitin, flucytosine, other drugs such as aspirin, cimetidine, probenecid, trimethoprim Conditions that decrease creatinine: advanced age, cachexia, liver disease **With ESRD an average of creatinine clearance and urea clearance is more accurate than creatinine clearance alone** If creatinine clearance is reduced renal failure is a possibility Complications of renal cystic dz = pain, hematuira, renal infections, renal stones, HTN, cerecral aneurysms Complications of chronic renal dz = HTn or hypOten, castrointestinal hemorrhage, Adult respiratory distress syndrome Intravenous Pyelogram Indications: (X-Ray with contrast) Functional exam of the kidney to see how fast it can filter Used as an anatomical exam Used to evaluate hematuria, urolithiasis/ureteral stones, renal cystic dz, drug induce nephropathy Significance of abnormal findings and complications: With renal cystic dz there are filling defects With drug induced nephropathy there is egg in cup focal necorsis, lobster claw calyces, scarring, sloughing and calcification of papilla and renal stones. Complication of IVP = one of the biggest causes of renal failure due to toxicity of the contrast used in the procedure Cystoscopy Indications: Used to assess for bladder or urethral neoplasm, benign prostatic enlargement, and radiation or chemical cystitis Bladder Vesical stones Significance of abnormal findings and complications: With vesical stones there can be a back pressure develop due to obstruction Ultrasound of the Kidneys, bladder & prostate Indications: Hematuria,

Significance of abnormal findings and complications: No specifics with this. Urine culture and sensitivity Indications: Cystitis, Pyelonephritis Prostatitis, Significance of abnormal findings and complications: Demonstrates infecting organism (culture is NOT requires for an acute uncomplicated episode in a young woman) Renal biopsy Indications: Unexplained acute renal failure or chronic kidney dz Acute nephritic syndrome Unexplained proteinuria and hematuria Previously identified and treated lesions to plan future therapy Systemic dz associated with kidney dysfunction such as SLE, Goodpasture syndrome, Wegener granulomatosis Suspected transplant rejection Guide to treatment Used to determine subtypes in nephritic syndrome Significance of abnormal findings and complications: Contraindications: solitary or ectopic kidney (exception transplant allograft), horseshoe kidney, uncorrected HTN, renal infection, renal neoplasm, hydronephrosis, ESRD, congenital anomalies, multiple cysts, or uncooperative pt After biopsy = nearly all pt have hematuria After percutaneous kidney biopsies a blood transfusion will be necessary due to significant blood loss and more than half of the pt will have a small hematoma Nephrectomy and mortality (0.06-0.08% chance)

Objective #2 State the usual etiologies, presentation, associated lab findings by etiology & management for: A. Hematuria B. Dysuria C. Proteinurea Hematuria Essentials to diagnosis o May be either grossly visible or microscopic; both require evaluation o The upper urinary tract should be imaged, and cystoscopy should be performed if there is hematuria in the absence of infection Etiologies o Upper tract sources (kidneys & ureters) 10% of cases 40% are stone diseases 20% renal disease (medullary sponge kidney, glomerulonephritis, papillary necrosis) 10% renal cell carcinoma 5% urothelial cell carcinoma of the ureter or renal pelvis Drug ingestion and associated medical problems may provide diagnostic clues Analgesic use= papillary necrosis Cyclophosphamide= chemical cystitis Abx= interstitial nephritis DM, sickle cell trait or disease= papillary necrosis o In the absence of infection, gross hematuria from a lower tract infection is most commonly from urothelial cell carcinoma of the bladder o Microscopic hematuria in the male is most commonly from benign prostatic hyperplasia

Hematuria in pts receiving anticoagulation therapy cannot be ascribed to the anticoagulation; a complete evaluation is warranted o Hematuria may be macroscopic or microscopic Microscopic is defined as excretion of >3 RBCs per high-power field in a centrifuged specimen o Common causes Glomerular Glomerulonephritis Medications (esp. NSAIDs) Hemolytic-uremic syndrome Strenuous exercise Non-glomerular Renal tumors Cystic kidney disease Papillary necrosis o Often medications Renal stones Infections Trauma Indewelling catheters Presentation o If gross hematuria occurs, a description of the timing (initial, terminal, total) may provide a clue to the localization of disease. o Associated symptoms should be investigated renal colic, irritative voiding symptoms, constitutional symptoms o Physical examination should emphasize signs of systemic disease fever, rash, lymphadenopathy, abdominal or pelvic masses & signs of medical kidney disease (hypertension, volume overload) o Urologic evaluation may demonstrate an enlarged prostate, flank mass, or urethral disease o Severe dehydration- renal vein thrombosis o Peripheral edema- nephritic syndrome, vasculitis o Cardiovascular MI- renal artery disease HTN- glomerulosclerosis o Abdomen Bruit- renal artery disase Enlarged prostate- prostatitis, prostatic hypertrophy Flank tenderness- renal stone, infarction Lab findings by etiology o Initial laboratory investigations include a urinalysis and urine culture o Proteinuria and casts suggest renal origin o Irritative voiding symptoms, bacteriuria, and a positive urine culture in the female suggest urinary tract infection, but follow-up urinalysis is important after treatment to ensure resolution of the hematuria o Further evaluation includes urinary cytology to assist in the diagnosis of bladder neoplasm Management o In the absence of infection, hematuria (either gross or microscopic) requires evaluation o Referral to nephrologist or pts with Significant proteinurea (>1g/day) Red cell casts or a predominance of dysmorphic red cells Renal insufficiency

Dysuria Etiologies o Women Urethritis, cystitis (all types) & pyelonephritis Vaginitis o Men Urethritis, cystitis (all types) and pyelonephritis

Prostatitis Obstruction (prostatic hypertrophy) Arthritis syndromes (reiters syndrome, bechets syndrome)

Presentation o The sensation of pain or burning on urination Lab findings by etiology Management Proteinurea(From the article & her notes only, she said not to refer to CMDT for proteinurea) Etiologies o Normal urinary proteins include: albumin, serum globulins, and proteins secreted by the nephron o Proteinurea is defined as the excretion of more than 150mg per day (10 to 20mg per dL) & is the hallmark of renal disease o Microalbuminuria is defined as the excretion of 30-150mg of protein per day and is a sign of early renal disease, esp. in diabetic pts o Can be classified as transient or persistent o See table 5, p.1157 of the article for a complete list of common causes o The majority of proteinuria is benign, this is termed functional Dehydration, emotional stress, most acute illnesses or fevers, inflammatory processes, orthostatic disorders, intense activity, heat injuries Presentation o Transient A temporary change in glomerular hemodynamics causes the protein excess Folloes a benign, self-limited course o Orthostatic (postural) proteinuria is a benign condition that can result from prolonged standing; it is confirmed by obtaining a negative U/A result after 8 hours of recumbancy o Persistent Divided into 3 general categories: 1. Glomerular 2. Tubular 3. Overflow Glomerular (most common) o Albumin is the primary urinary protein o Increased glomerular capillary permeability to proteins Tubular o Results when malfunctioning tubule cells no longer metabolize or reabsorb normally filtered protein o Low-molecular weight proteins predominate over albumin and rarely exceed 2g per day Overflow o Low-molecular weight proteins overwhelm the ability of the tubules to reabsorb filtered proteins (hemoglobin, myoglobin, or immunoglobins) Further evaluation inclused determination of 24-hour urinary protein excretion or spot urinary proteincreatinine ratio, microscopic examination of the urinary sediment, urinary protein electrophoresis, and assessment of renal function Lab findings by etiology o Dipstick analysis is used for screening and a crude estimate of quantity False positives can occur from a variety of conditions It preferentially detects the larger proteins, such as albumin o Twenty-four hour urine collection is used for a more specific quantitative evaluation o Microscopic analysis of the patient with persistent proteinuria is done to assess for etiology by demonstration of abnormal by-products Leukocytes or leukocytes casts with bacteria occur in Infections Dysmorphic erythrocytes occur in kidney disease, normal erythrocytes are lower system. Erythrocyte casts demonstrate glomerular disease Eosinophiluria demonstrates drug-induced interstitial nephritis Waxy, granular or cellular casts appear in advanced renal disease of any cause. Hyaline casts appear with dehydration or diuretic therapy o Adults with proteinuria > 2 grams per twenty-four hours requires measurement of creatinine clearance o Creatinine Clearance If creatinine clearance is reduced renal failure is a major possibility and therefore a nephrology consultation is in order

If creatinine clearance is normal, and the patient has a clear diagnosis such as diabetes or congestive heart failure, treat the underlying disease

Management o Transient proteinuria Treat any underlying conditions and reassure o Orthostatic proteinuria Blood pressure measurement and repeat urinalysis every 1-2 years if under 30 years old and < 2 g/day o Isolated Proteinuria If the patient is over 30 years old or have proteinuria not associated with posture, they are higher risk and needs blood pressure measurements, urinalysis and creatinine clearance every 6 months. o If they have more than 2 grams per day this is more serious and needs a nephrology consultation.

Objective #3 State the etiology, clinical presentation, evaluation, lab and imaging findings, management and complications of the following types of urinary system infections: a. Acute Pyelonephritis d. Bacterial Prostatitis (all types) b. Acute Cystitis e. Nonbacterial Prostatitis c. Interstitial Cystitis Acute Pyelonephritis Essentials of Diagnosis o Fever o Flank Pain o Irritative voiding sx o Positive urine culture Etiology o Acute pyelonephritis is an infectious inflammatory disease involving the kidney parenchyma and renal pelvis o Gram-negative bacteria are the most common causative agents including E coli, Proteus, Klebsiella, Enterobacter, and Pseudomonas o Gram-positive bacteria are less commonly seen o The infection usually ascends from the lower urinary tract with the exception of S aureus, which usually is spread by a hematogenous route Clinical Presentation o Symptoms include fever, flank pain, shaking chills, and irritative voiding symptoms (urgency, frequency, dysuria) o Associated nausea and vomiting and diarrhea are common o Signs include fever and tachycardia o Costovertebral angle tenderness is usually pronounced Evaluation Lab findings o CBC shows leukocytosis and a left shift o Urinalysis shows pyuria, bacteriuria, and varying degrees of hematuria o White cell casts may be seen o Urine culture demonstrates heavy growth of the offending agent, and blood culture may also be positive Imaging Findings o In complicated pyelonephritis, renal ultrasound may show hydronephrosis from a stone or other source of obstruction Management o Severe infections or complicating factors require hospital admission o Urine and blood cultures are obtained to identify the causative agent and to determine antimicrobial sensitivity o IV ampicillin and an aminoglycoside are initiated prior to obtaining sensitivity results o In the outpatient setting, quinolones or nitrofurantoin may be initiated o Fevers may persist for up to 72 hours failure to respond warrants imaging (CT or ultrasound) to exclude complicating factors that may require intervention o Catheter drainage may be necessary in the face of urinary retention and nephrostomy drainage if there is ureteral obstruction o In inpatients, IV antibiotics are continued for 24 hours after the fever resolves, and oral antibiotics are then given to complete a 14-day course of therapy

o Follow-up urine cultures are mandatory following the completion of treatment Complications o Sepsis with shock o In diabetic patients, emphysematous pyelonephritis resulting from gas-producing organisms may be lifethreatening if not adequately treated o Healthy adults usually recover complete kidney function, yet if coexistent kidney disease is present, scarring or chronic pyelonephritis may result o Inadequate therapy could result in abscess formation Acute Cystitis Essentials of Diagnosis o Irritative voiding sx o Pt. usually afebrile o + urine culture, blood cultures may also be + Etiology o Acute cystitis is an infection of the bladder most commonly due to the coliform bacteria (especially Escherichia coli) and occasionally gram-positive bacteria (enterococci) o The route of infection is typically ascending from the urethra o Viral cystitis due to adenovirus is sometimes seen in children but rare in adults o Cystitis in men is rare and implies a pathologic process such as infected stones, prostatitis, or chronic urinary retention requiring further investigation Clinical Presentation o Irritative voiding symptoms (frequency, urgency, dysuria) and suprapubic discomfort are common o Women may experience gross hematuria, and symptoms in women may often appear following sexual intercourse o PE may elicit suprapubic tenderness, but examination is often unremarkable o Systemic toxicity is absent Evaluation Lab findings o Urinalysis shows pyuria and bacteriuria and varying degrees of hematuria o The degree of pyuria and bacteriuria does not necessarily correlate with the severity of symptoms o Urine culture is positive for the offending organism Imaging Findings o Because uncomplicated cystitis is rare in men, clarification of the underlying problem with appropriate investigations, such as abdominal ultrasonography or cystoscopy (or both) is warranted o F/U imaging using CT scanning is warranted if pyelonephritis, recurrent infections, or anatomic abnormalities are suspected Management Infections typically respond rapidly to therapy, and failure to respond suggests resistance to the selected drug or anatomic abnormalities requiring further investigation Uncomplicated cystitis in women can be treated with short-term abx therapy Fluoroquinolones and nitrofurantoin are the drugs of choice for uncomplicated cystitis In men, uncomplicated UTI is rare & the duration of abx therapy depends on the underlying etiology Hot sitz baths or urinary analgesics may provide symptomatic relief Complications Interstitial Cystitis Essentials of Diagnosis o Pain with a full bladder or urinary urgency o Submucosal petechiae or ulcers on cystoscopic examination o Dx of exclusion Etiology o Characterized by pain with bladder filling that is relieved by emptying and is often associated with urgency and frequency o This is a diagnosis of exclusion, and patients must have a negative urine culture and cytology and no other obvious cause such as radiation cystitis, chemical cystitis (cyclophosphamide), vaginitis, urethral diverticulum, or genital herpes o Up to 40% of patients referred to urologists for interstitial cystitis may actually be found to have a different diagnosis after careful evaluation o Most patients are women, with a mean age of 40 years at onset

Patients with interstitial cystitis are more likely to report bladder problems in childhood, and there appears to be a higher prevalence in white and Jewish women o Up to 50% of patients may experience spontaneous remission of symptoms, with a mean duration of 8 months without treatment o Etiology is unknown, and it is most likely not a single disease but rather several diseases with similar symptoms o Associated diseases include severe allergies, IBS, or inflammatory bowel disease o Theories regarding the cause of interstitial cystitis include increased epithelial permeability, neurogenic causes (sensory nervous system abnormalities), and autoimmunity Clinical Presentation o Most common sx: Pain with bladder filling that is relieved with urination or urgency, frequency, & nocturia o Exposures such as pelvic radiation or prior cyclophosphamide should be inquired about Evaluation o Examination should exclude genital herpes, vaginitis, or a urethral diverticulum Lab findings o Urinalysis and urine culture are obtained to exclude infectious causes o Urinary cytology is obtained to exclude bladder malignancy o Urodynamic testing assesses bladder sensation and compliance and excludes detrusor instability Imaging Findings o The bladder is distended with fluid to detect glomerulations (submucosal hemorrhage), which may or may not be present o Biopsy should be performed to exclude other causes such as carcinoma, eosinophilic cystitis, and tuberculous cystitis o The presence of submucosal mast cells is not needed to make the diagnosis of interstitial cystitis Management o There is no cure, but most patients achieve symptomatic relief from one of several approaches Hydrodistention (which is usually done as part of the diagnostic evaluation) Approximately 2030% of patients will notice symptomatic improvement following this maneuver. Measurement of bladder capacity during hydrodistention, since patients with very small bladder capacities (< 200 mL) are unlikely to respond to medical therapy. o Amitriptyline is often used as first-line medical therapy in patients with interstitial cystitis Complications

Bacterial Prostatitis (all types) Acute Bacterial Prostatitis o Essentials of Diagnosis Fever Irritative voiding sx Perineal or suprapubic pain; exquisite tenderness common on rectal exam Positive urine culture o Etiology Acute bacterial prostatitis is usually caused by gram-negative rods (esp. E coli and Pseudomonas) and less commonly by gram-positive organisms enterococci) The most likely routes of infection include ascent up the urethra and reflux of infected urine into the prostatic ducts Lymphatic and hematogenous routes are rare o Clinical Presentation Perineal, sacral, or suprapubic pain, fever, and irritative voiding complaints are common. Varying degrees of obstructive symptoms may occur as the acutely inflamed prostate swells, which may lead to urinary retention High fevers and a warm and often exquisitely tender prostate are detected on examination Care should be taken in performing a gentle rectal examination, as vigorous manipulations may result in septicemia Prostatic massage is contraindicated o Evaluation o Lab findings CBC shows leukocytosis and a left shift Urinalysis shows pyuria, bacteriuria, and varying degrees of hematuria

Urine cultures will demonstrate the offending pathogen Imaging Findings Management Hospitalization may be required, and parenteral antibiotics should be initiated until organism sensitivities are available After the patient is afebrile for 2448 hours, oral antibiotics are used to complete 46 weeks of therapy If urinary retention develops, urethral catheterization or instrumentation is contraindicated, and a percutaneous suprapubic tube is required F/U urine culture and exam of prostatic secretions should be performed after the completion of therapy to ensure eradication o Complications With effective treatment, chronic bacterial prostatitis is rare Chronic Bacterial Prostatitis o Essentials of Diagnosis Irritating voiding sx Perineal or suprapubic discomfort, often dull & poorly localized Positive expressed prostatic secretions & culture o Etiology Although chronic bacterial prostatitis may evolve from acute bacterial prostatitis, many men have no history of acute infection Gram-negative rods are the most common etiologic agents only one gram-positive organism (Enterococcus) is associated with chronic infection Routes of infection are the same as for acute infection o Clinical Presentation Clinical manifestations are variable Some patients are asymptomatic, but most have varying degrees of irritative voiding symptoms Low back and perineal pain are not uncommon Many pts report a history of UTIs PE is often unremarkable, though the prostate may feel normal, boggy, or indurated o Evaluation o Lab findings Urinalysis is normal unless a secondary cystitis is present Expressed prostatic secretions demonstrate increased numbers of leukocytes However, this finding is consistent with inflammation and is not diagnostic of bacterial prostatitis Leukocyte and bacterial counts from expressed prostatic secretions do not correlate with severity of symptoms Culture of the secretions or the postprostatic massage urine specimen is necessary to make the diagnosis o Imaging Findings Imaging tests are not necessary, though pelvic radiographs or transrectal ultrasound may demonstrate prostatic calculi o Management Few antimicrobial agents attain therapeutic intraprostatic levels in the absence of acute inflammation Trimethoprim-sulfamethoxazole is associated with the best cure rates Optimal duration of therapy remains controversial, ranging from 6 to 12 weeks Symptomatic relief may be provided by anti-inflammatory agents and hot sitz baths o Complications Chronic bacterial prostatitis is difficult to cure, but its symptoms and tendency to cause recurrent urinary tract infections can be controlled by suppressive antibiotic therapy Nonbacterial Prostatitis Essentials of Diagnosis o Irritative voiding sx o Perineal or suprapubic discomfort, similar to that of chronic bacterial prostatitis o + Expressed prostatic secretions, but culture is negative Etiology o The most common of the prostatitis syndromes, and its cause is unknown o Speculation implicates chlamydiae, mycoplasmas, ureaplasmas, and viruses, but no substantial proof exists o Sometimes nonbacterial prostatitis may represent a noninfectious inflammatory or autoimmune disorder o o

o Because the cause of nonbacterial prostatitis remains unknown, the diagnosis is usually one of exclusion Clinical Presentation o The clinical presentation is identical to that of chronic bacterial prostatitis, however, no history of UTI is present Evaluation Lab findings o Increased numbers of leukocytes are seen on expressed prostatic secretions o All cultures are negative Imaging Findings Management o Because of the uncertainty regarding the etiology of nonbacterial prostatitis, a trial of antimicrobial therapy directed against Ureaplasma, Mycoplasma, or Chlamydia is warranted o Symptomatic relief may be obtained with anti-inflammatory agents or sitz baths o Dietary restrictions are not necessary unless the patient relates a history of symptom exacerbation by certain substances such as alcohol, caffeine, and perhaps certain foods Complications

Objective 4 State the etiology, clinical presentation, pathophysiology, evaluation, lab and imaging findings, management and complications of the following types of urinary obstruction or stasis: a) urinary stone disease b) benign prostatic hypertrophy Urinary Stone Disease p. 833 Essentials of Diagnosis Flank pain Nausea and vomiting Identification on noncontrast CT Etiology & Men > women rd th Pathophysiology Most commonly presents in 3 or 4 decade th th Men = women in 6 and 7 decade 5 major types of stones: Calcium oxalate Calcium phosphate Struvite Uric acid Cystine Stone formation requires saturated urine that is dependent on: pH Ionic strength Solute concentration Complexation Geographic factors: areas of high humidity and elevated temperatures Incidence is greatest during hot summer months Diet and fluid intake: Increased sodium intake will: Increase sodium and calcium excretion Increase monosodium urate saturation Increase relative saturation of calcium phosphate Decrease urinary citrate excretion All of these factors encourage stone growth Increased protein load can also increase calcium, oxalate, and uric acid excretion and decrease urinary citrate excretion Carbs and fats have no impact on stone formation Bran decreases urinary calcium Persons in sedentary occupations have higher incidence than manual laborers Genetic factors may contribute: cystinuria & distal renal tubular acidosis Clinical Presentation Usually present with colic Pain occurs suddenly and may awaken patients from sleep

Evaluation

Lab & Imaging Findings

Management

Localized to the flank, severe, may be associated with N/V May be episodic and radiate anteriorly over the abdomen Pts constantly moving As the stone progresses down the ureter pain may be referred to the ipsilateral testes or labium If lodged at the ureterovesicular junction: urinary urgency and frequency Stone size does not correlate with severity of symptoms Stone analysis should be performed on recovered stones st 1 time stone-formers: blood screening for: calcium phosphate electrolytes uric acid Recurrent stone-formers or those with a family hx: 24-hour urine collection volume urinary pH calcium uric acid oxalate phosphate sodium citrate Serum parathyroid and calcium load tests can be done Urinalysis Microscopic or gross hematuria (but the absence of microhematuria does not exclude urinary stones) pH is a valuable clue to the cause of the stone persistent pH below 5.5 is suggestive of uric acid or cystine stones a pH above 7.2 is suggestive of a struvite infection stone Plain film of the abdomen and renal ultrasound will diagnose most stones st CT scan is 1 line for evaluating flank pain (all stones are visible on CT) Medical tx One must attempt to achieve a stone-free status Small stone fragments may serve as a nidus for future stone development Increase fluid intake During meals, 2 hours after each meal, prior to going to sleep, and throughout the night Tx underlying reason why stones formed Calcium nephrolithiasis o Hypercalciuric Cellulose phosphate Thiazides o Hyperuricosuric Purine dietary restrictions Allopurinol o Hyperoxaluric Oral calcium supplements o Hypocitraturic Potassium citrate supplements Uric acid calculi o Potassium citrate Struvite calculi o Appropriate abx Cystine calculi o Increased fluid intake

o o

Monitor urine pH Penicillamine and tiopronin

Complications

Surgical tx Ureteral stones Stones < 6mm will usually pass spontaneously Conservative observation with appropriate pain meds is appropriate for the first 6 weeks Distal ureteral stones o Ureteroscopic stone extraction o In situ extracorpeal shock wave lithotripsy (SWL) Proximal and midureteral stones o SWL or ureteroscopy Renal stones Those that present without pain, urinary tract infections, or obstruction need not be treated Growing calculi or symptomatic pts should be treated o < 2 cm: SWL o Larger diameter: percutaneous nephrolithotomy Unknown

Benign Prostatic Hyperplasia p. 842 Essentials of Diagnosis Obstructive or irritative voiding symptoms May have enlarged prostate or rectal examination Absence of urinary tract infection, neurologic disorder, stricture disease, prostatic or bladder malignancy Etiology & Most common benign tumor in men Pathophysiology Incidence is age related Symptoms of prostatic obstruction are also age related 55 yo: 25% report voiding sx 75 yo: 50% report a decrease in force and caliber of stream Risk factors poorly understood Heritable form most likely autosomal dominant Clinical Presentation Obstructive complaints Hesitancy Decreased force and caliber of stream Sensation of incomplete bladder emptying Double voiding: urinating a second time within 2 hours Straining to urinate Postvoid dribbling Irritative symptoms Urgency Frequency Nocturia Evaluation AUA symptom index: single most important tool for evaluation Should be calculated in all patients before starting therapy Score can range from 0-35 Detailed hx focusing on the urinary tract to rule out: UTI, neurogenic bladder, or urethral stricture Physical exam, DRE, neurologic exam Prostate size does not correlate with the severity of symptoms or degree of obstruction Benign: smooth, firm, elastic enlargement of the prostate Cancer: induration Lower abdomen exam should be done to assess the bladder Lab & Imaging Urinalysis Findings Serum PSA Upper tract imaging (IVU, CT, ultrasound) Flow rates, postvoid residual urine determination, pressure-flow studies

Management

Complications Objective 5

optional Watchful waiting: Pts with mild symptoms (AUA score 0-7) Some men undergo spontaneous improvement or resolution of sx Medical Therapy a-blockers prazosin, terazosin, doxazosin 5a-reductase inhibitors finasteride Combination therapy Phytotherapy The use of plants or plant extracts for medical purposes Saw palmetto berry, bark of pygeum africanum, etc. (p. 846 if youre really interested) Surgery: refractory urinary retention, large bladder diverticula, recurrent UTIs, recurrent gross hematuria, bladder stones, or renal insufficiency Transurethral resection of the prostate (TURP) Transurethral incision of the prostate (TUIP) Open simple prostatectomy Laser therapy Transurethral needle ablation of the prostate (TUNA) Transurethral electrovaporization of the prostate Hyperthermia Unknown

Diabetic Nephropathy Essentials of Diagnosis: *prior evidence of diabetes mellitus * albuminuria (micro or macroscopic) not due to another renal disorder *signs of diabetic nephropathy on renal biopsy, if done. *other end organ damage common, such as diabetic retinopathy, but not required. History -Most common cause of ESRD in U.S. -ESRD is much more likely to develop in persons with type 1 diabetes mellitus -High risk pts: males, AA, Native Americans

Clinical Presentation & Examination

-Pts will usually have enlarged kidneys due to cellular hypertrophy and proliferation. -The most common lesion is diffuse glomerulosclerosis -Nodular glomerulosclerosis is PATHOGNOMONIC!! (aka Kimmelstiel-Wilson nodules) -Diabetic retinopathy is often present -Initial screening of diabetics should always include urine examination for microalbuminuria. -24 hour urine collection is the accepted standard measure. -Can also test via an early morning spot urine albumin or albumin-creatinine ratio. (more than 30mg-1g is abnormal) -Pts prone to other renal dzs such as papillary necrosis, chronic intersititial nephritis, and type 4 renal tubular acidosis -Pts more susceptible to acute renal failure from contrast material -poor prognosis once dialysis has begun. -Aggressive tx during the onset of microalbuminuria (before onset of clinical proteinuria) -Strict glycemic control -tx of HTN with ACE and ARBs

Work-up

Complications

Prognosis Management

Analgesic Nephropathy p 822 **dont need to worry about other types of drug induced nephropathy** History -Most commonly seen in pts who ingest a large amt of analgesic combinations; esp for chronic HA, muscular pains, and arthritis. -Causative drugs: phenacetin, paracetamol, aspirin, and NSAIDs. -Ingestion of 1g/d X 3 yrs si the typical amt needed for renal dysfxn. S/Sx -~50% present in acute renal failure -flank pain, hematuria, fever -+/- polyuria w/ dehydration -anemia -edema (if Na+ retention) -Nephrotic syndrome if dz is advanced -Tubulointerstitial inflammation and papillary necrosis are seen on pathologic examination. -Papillary tip and inner medullary concentrations of some analgesic are 10 fold higher than in the renal cortex. -Aspirin and other NSAIDs may worsen the damage by decreasing medullary blood flow and decreasing glutathione levels. -May have sloughed papillae in the urine. -Ring Shadow sign at the papillary tip -broad waxy casts are often present. -Hyperkalemic -Hyperchloremic renal tubular acidosis is characteristic -Pts can exhibit hematuria, mild proteinuria, polyuria, anemia, and sterile pyuria -Na+ retention and edema -acute renal insufficiency or failure -Nephortic syndrome with interstitial nephritis

Clinical Presentation & Examination

Work-up

Labs

Complications Prognosis Management

-Papillary necrosis -Urinalysis -Renal Fxn test -IVP -Radiographs -Urinalysis: hematuria, low levels of proteinuria and proteinaceous casts. Neutrophils and cellular debris -Renal Fxn test: Elevated BUN and Creatinine -Radiographs: non-specific, KUB may show small flecks of Ca++ in the kidney (dystrophic calcification) -IVP: egg in cup focal necrosis, lobster claw calyces, scarring, sloughing and calcification of papilla and renal stones. -HTN, obstruction from necrotic papillae or calcium, anemia, frequent UTIs, medication toxicities

-W/drawl from analgesics -stablization or improvement of renal fxn may occur if significant interstitial fibrosis is not present. -Hydration during exposure of analgesics may help. -Mostly supportive. -Dialysis if severe poisoning is present

Objective 6 State the etiology, clinical presentation, laboratory and imaging findings, management and complications of: a) acute tubular necrosis b) interstitial nephritis c) glomerulonephritis (both types) Acute Tubular Necrosis p. 799 Essentials of Diagnosis Acute kidney injury Clinical scenario consistent with diagnosis (ischemic or toxic insult) Urine sediment with pigmented granular casts and renal tubular epithelial cells is pathognomonic but not essential Etiology & Acute renal failure due to tubular damage Pathophysiology Accounts for 85% on intrinsic acute renal failure 2 causes: ischemia and nephrotoxin exposure Ischemia: causes damage from states of low perfusion and is often preceded by a state of prerenal azotemia inadequate GFR and renal blood flow to maintain parenchymal cellular formation prolonged hypotension or hypoxemia (dehydration, shock, sepsis) Major surgical procedures can involve prolonged periods of hypoperfusion Nephrotoxins: exogenous > endogenous Exogenous: anminoglycosides, amphotericin B, radiographic contrast material, NSAIDs, ACE inhibitors, cyclosporine, heavy metals Endogenous: myoglobinuria (from rhabdomyolysis), hemoglobin, hyperuricemia, Bence Jones protien (multiple myeloma) Clinical Presentation Same as acute renal failure Evaluation/ Urinalysis Lab & Imaging Urine may be brown Findings May show pigmented granular casts or muddy brown casts Renal tubular epithelial cells and epithelial cell casts may also be present Hypekalemia and hyperphosphatemia common Management Preventative measures: to avoid volume overload and hyperkalemia Loop diuretics (furosemide) When to refer: Pt with acute renal failure should be referred to a nephrologist when the etiology is unclear or renal fxn continues to worsen despite intervention If fluid, electrolyte, and acid-base abnormalities are recalcitrant to interventions Nephrology referral improves outcome in acute renal failure o When to admit: When pt has signs or symptoms of acute renal failure that require immediate intervention

Complications

such as IV fluids, dialytic therapy, or a team approach cannot be coordinated as an outpatient Unknown

Interstitial Nephritis p. 801 Essentials of Diagnosis: Fever Transient maculopapular rash Acute renal insufficiency Pyuria (including eosinophilia), whit blood cell casts, and hematuria Etiology & 10-15% of intrinsic renal failure Pathophysiology Interstitial inflammatory response with edema and possible tubular damage T lymphocytes can cause direct cytotoxicity or release lymphokines that recruit monocytes and inflammatory cells Drugs account for 70% of cases PCN, cephalosporins, sulfonamides and sulfonamide-containing diuretics, NSADIs, rifampin, phenytoin, allopurinol, PPIs Can also be caused by: infectious diseases, immunologic disorders, or idiopathic condition Infections: streptococcal infxn, leptospirosis, cytomegalovirus, histoplasmosis, Rocky Mountain spotted fever Immunologic: glomerulonephritis, systemic lupus erythematosus, Sjogren syndrome, sarcoidosis, cryoglobulinemia Clinical Presentation Fever Rash Arthralgias Evaluation Unknown Lab & Imaging Peripheral blood eosinophilia Findings Urinalysis Red cells, white cells, and white cell casts Modest proteinuria (usually NSAID induced) Eosinophiluria Management Supportive measures and removal of the inciting agent Short course of corticosteroids may be used Prednisone or methylprednisolone Complications None. Good prognosis. Rarely, some progress to ESRD Glomerulonephritis Acute Glomerulonephritis p. 802 Essentials of Diagnosis: Hematuria, dysmorphic red cells, red cell casts, and mild proteinuria Dependent edema and hypertension Acute renal insufficiency Etiology & Uncommon cause of renal failure (5%) Pathophysiology Inflammatory glomerular lesions Mesangioproliferative, focal and diffuse proliferative, and crescentric lesions Categorization of acute glomerulonephritis can be done by serologic analysis Cytoplasmic antibodies Moderate antigen excess over antibody production occurs Causes include: o IgA nephropathy (Bergers disease) o Peri-infectious or post-infectious glomerulonephritis o Endocarditis o Lupus nephritis o Cryoglobulinemic glomerulonephritis (hep C virus)

Clinical Presentation Evaluation

Lab & Imaging Findings

Management Complications Postinfectious Glomerulonephritis p. 811

o Membranoproliferative glomerulonephritis Anti-GBM antibodies Confined to kidneys or associated with pulmonary hemorrhage Goodpasture syndrome autoantibodies aimed against type IV collagen in the GBM rather than to immune complex deposition Other immune markers of disease Pauci-immune acute glomerulopnephritis o Due to cell-mediated immune processes Vascular causes o Malignant HTN o Thrombotic microangiopathies (hemolytic uremic syndrome & TTP) Pts are usually hypertensive and edematous Edema first presents in body parts with low tissue tension (periorbital and scrotal regions) 24 hour urine for protein excretion and creatinine clearance however, in cases of rapidly changing serum creatinine values, urinary creatinine clearance is an unreliable marker of GFR Further tests: Complement levels (C3, CA, CH50) ASO titer Anti-GBM antibody levels ANCAs Antinuclear antibody titers Cryoglobulins Hepatitis serologies Blood cultures Renal ultrasound Renal biopsy (occasionally) Urinalysis Hematuria Moderate proteinuria (usually < 3 g/d) Cellular elements: red cells, red cell casts, and white cells Red cell casts are specific for glomerulonephritis High dose corticosteroids and cytotoxic agents (cyclophosphamide)

Essentials of Diagnosis: Proteinuria Glomerular hematuria Symptoms 1-3 weeks after infection Etiology & Most often due to infxn with nephritogenic group A B-hemolytic strep, especially type 12 Pathophysiology Onset occurs within 1-3 weeks after infection (average 7-10 days) Other causes: Bacteremic states (systemic S. aureus ifxn), infective endocarditis, shunt infections, heptatis B or C, cytomegalovirus, mononucleosis, coccidioidmycosis, malaria, toxoplasmosis Clinical Presentation Pt is oliguric, edematous, and variably hypertensive & Evaluation Lab & Imaging Serum complement levels low Findings ASO titers can be high in strep infxn Urinalysis Cola-colored urine Red blood cells and red cell casts Proteinuria < 3.5 g/d Immunofluorescence IgG and C3 in granular pattern in the capillary basement membrane

Management

Complications

Electron microscopy Large, dense subepithelial deposits or humps Supportive Appropriate abx should be used Antihypertensives, salt restriction, and diuretics if needed Adults prone to crescent formation and chronic renal insufficiency Rapidly progressive glomerulonephritis in < 5% Smaller percentage will progress to ESRD

Objective 7 State the etiology, risk factors, clinical presentation, laboratory and imaging findings, management and complications of Nephritic and Nephrotic Syndrome. Nephritic Syndrome Essentials of Diagnosis: Edema Hypertension Hematuria (with or without dysmorphic red cells, red blood cell casts) Etiology Due to an acute inflammatory process in the glomeruli that causes renal dysfunction over days to weeks and may or may not resolve. Subtypes (based on how dz presents) o If severe (rapidly progressive glomerulonephritis)= 50% loss of nephron fxn over just a few weeks. Can cause permanent damage to glomeruli (necrosis, fibrosis, etc.). RPGN I: Anti-GBM dz (Goodpastures, Masugi, etc.) (20% of RPGN cases) The patient makes antibodies against an antigen uniformly distributed along the GBM. RPGN II: Severe immune complex disease (any of these diseases can produce RPGN if it is severe enough) o Acute (postinfectious) glomerulonephritis = produces the nephritic syndrome two weeks following a respiratory or skin infection with streptococcus. Commonly appears after pharyngitis or impetigo (1-3 weeks post-infection; average 710 days) o Prolonged inflammation = chronic glomerulonephritis with persistent renal abnormalities that lead to ESRD. Mild edema in low tissue pressure areas (periorbital, scrotal area). Hypertension due to volume overload not vasoactive substances (angiotensin II - this substances are low in the body). Oliguria Proteinuria (3-5g/day) RPGN I: Patients experience hemoptysis, dyspnea, and possible respiratory failure. RPGN II: o Causes: Lupus vasculitis, Henoch-Schonlein (IgA nephropathy = gross hematuria turning to cocacola colored urine, and purpura on the skin), and the vasculitis of subacute bacterial endocarditis. RPGN III: the vasculitis syndromes o Systemic inflammatory disease symptoms, including fever, malaise, and weight loss. Laboratory Findings: No serum chemistries characteristic of nephritic syndrome. Special tests: complement levels, antinuclear antibodies (ANA), cryoglobulins, hepatitis serologies, ANCA, anti-GBM antibodies (diagnostic), antistreptolysin O (ASO) titers, and C3 nephritic factor. Urinalysis: Hematuria: dysmorphic red blood cells and red blood cell casts. Proteinuria (3-5 g/day) **If you place the patient in the lordotic position for an hour it increases the sensitivity for

Clinical Presentation & Examination

Work-up

Complications Management

finding red cell casts in the next urine sample. Biopsy: Do if no contraindications (bleeding disorder, thrombocytopenia, uncontrolled HTN) When > 50% of the glomeruli contain crescents = rapidly progressive glomerulonephritis Type of dz can be categorized based on the immunofluorescent pattern and appearance on electron microscopy from biopsy. Renal failure Reduction of HTN and fluid overload Salt and water restriction Diuretic therapy (cautiously; inpatient) Possibly dialysis Possibly corticosteroids and cytotoxic agents (if glomerular injury is present) Treatment of uremia Specific therapeutic maneuvers aimed at the underlying cause.

Nephrotic Syndrome Essentials of Diagnosis: Urine protein excretion > 3.5 g/1.73 m2 per 24 hours. Hypoalbuminemia (albumin < 3 g/dL). Peripheral edema. Etiology Indicates excessive permeability of the filtration membrane to plasma proteins. 1/3 of patients with nephrotic syndrome have a systemic renal dz such as DM, amyloidosis, or systemic lupus erythematosus. Subtypes of nephritic syndrome (need biopsy for dx) Minimal change disease: seen in children or adults with Hodgkins lymphoma (good prognosis) Focal Segmental Glomerulosclerosis: seen in HIV, heroin abuse and chronic reflux nephropathy (poor prognosis) Membranous nephropathy: seen in carcinoma, Hodgkins lymphoma, lupus erythematosis Membranoproliferative Glomerulonephropathy

Clinical Presentation & Examination

Work-up

Heavy proteinuria (> 3.5 g/1.73 m per 24 hours) Frothy urine Hypoalbuminemia (<3.0g/dL) Mild cases = only small plasma proteins (albumin) will be lost, and the pt. has selective proteinuria. Severe cases = nonselective proteinuria. Severe generalized edema Peripheral edema due to sodium retention (HALLMARK) = when serum albumin < 3g/dL Usually lower extremities Dyspnea (due to the edema), pleural effusions and pulmonary edema Hyperlipidemia Periorbital edema and facial swelling (anascara) ascites and abdominal fullness Increased infection rate (due to loss of immunoglobulins and certain complement moieties in the urine) Urinalysis: Heavy proteinuria (adult > 3.5g/day) Frothy urine Hypoalbuminemia (<3.0gm/dL) Few casts in the urine sediment Addition of sulfosalicylic acid to urine sediment = abnormal paraproteins Oval fat bodies (if hyperlipidemia present) (grape clusters under ligh t microscopy and maltese crosses under polarized light) Renal biopsy: Performed in adults with new-onset idiopathic nephrotic syndrome if a primary renal dz that may require drug therapy (corticosteroids or cytotoxins) is suspected. Severely elevated creatinine levels indicates irreversible renal dz

Complications Management

Useful for prognosis and treatment of systemic renal dz Blood chemistries: Decreased serum albumin (<3g/dL) Total serum protein (< 6g/dL) Hyperlipidemia (the more protein excreted, the more frequency of hyperlipidemia increases) Increased hepatic production of lipids (cholesterol and apolipoprotein B) Decreased clearance of very low-density lipoproteins = hypertriglyceridemia Elevated erythrocyte sedimentation rate (due to increased levels of fibrinogen) Less common tests: complement levels, serum and urine protein electrophoresis, ANA, and serologic tests for hepatits. Decreased vitamin D, zinc, and copper (due to loss of binding proteins in the urine) Renal Failure Treatment in four parts (protein loss, edema, hyperlipidemia, and hypercoagulable state) Protein loss: the daily total protein intake should replace the daily urinary protein losses so as to avoid negative nitrogen balance. Ace inhibitors (can use ACEs despite compromised GFR as long as significant hyperkalemia does not occur and serum creatinine rises less than 30%, stabilizing over 2 months) Edema: Dietary salt restriction is essential! Usually requires diuretic therapy as well. (thiazide and loops) Hyperlipidemia: Dietary modification and exercise. Aggressive pharmacologic tx Hypercoagulable State: Pts with serum albumin < 2 g/dL can become hypercoagulable. Causes pt.s to be prone to renal vein thrombosis and other venous thromboemboli Anticoagulation therapy given for at least 3-6 months (in pt.s with evidence of thrombosis) Pts with renal vein thrombosis and recurrent thromboemboli = indefinite anticoagulation.

Objective 8 State the usual clinical presentation, laboratory and imaging findings, management and complications of the following Renal Cystic diseases: Polycystic Disease *Multiple cysts in bilateral kidneys; total number depends on age *Large, palpable kidneys on examination *Combination of HTN and abdominal mass suggestive of disease *FHx is compelling but not necessary *Chromosomal abnormalities present in some patients Among the most common hereditary diseases in the U.S. (1 in 800 live births) 50% of patients will have ESRD(end stage renal disease) 60yo Genes accounting for this d/o ADPKD1 on short arm of chromosome 16, and ADPKD2 on chromosome 4. PKD1 is worse. Autosomal dominant with variable penetrance Clinical Presentation Abdominal flank pain with microscopic or gross hematuria are present in most patients Hx of UTIs and nephrolithiasis is common (recurrent infections) + FHx in 75% of cases, and more than 50% of patients have HTN that may precede the clinical manifestations of the disease Large kidneys which may be palpable on the PE The combination of HTN + an abdominal mass should suggest the disease 40-50% have current hepatic cysts Pancreatic and splenic cysts occur also Hgb and Hct tend to be maintained as a result of EPO production by the cysts +/- palpable enlarged or nodular kidneys (more common as disease progresses) Hematuria, polyuria, nocturia. Late findings of renal insufficiency HA, N/V, weakness, weight loss Lab findings

U/A may show hematuria and mild proteinuria Anemia, Elevated BUN & Creatinine in serum Dilute urine late in disease If infected bacteriuria and pyuria Imaging In patients with PKD1, Ultrasonography confirms the Dx - 2+ cysts in Pts < 30yo (sensitivity is 88.5%). 2+ cysts in each kidney in Pts 30 -59 (sensitivity 100%). 4 or more in Pts 60 years or older is diagnostic of autosomal dominant polycystic kidney disease. If sonography is unclear, a CT scan is highly recommended and highly sensitive Management Fluid intake at 3000mL + low protein diet (0.5-0.75g/kg/d) is indicated as BUN rises. Avoiding caffeine may help prevent cysts. To treat the PAIN bed rest & analgesics are recommended. Cyst decompression (only if obstructing urine flow, otherwise not effective) can help with chronic pain. HEMATURIA typically resolves in 7d with bed rest an hydration RENAL INECTION CT scan can be helpful to diagnose. Bacterial cyst infxns are difficult to treat Abx with cystic penetration should be used, eg, fluoroquinolones, TMP-SMX, and chloramphenicol. Tx may be 2 weeks parenteral + long term oral therapy NEPHROLITHIASIS primarily calcium oxalate stones. Treat with hydration (2-3L/day) HTN should be treated aggressively because is will prolong the time to ESRD (diuretics should be used cautiously) CEREBRAL ANEURYSM elective surgery Supportive Tx in for renal function (dialysis in late stage due to renal insufficiency) Complications Abdominal or flank PAIN caused by infxn, bleeding into cysts and nephrolithiasis HEMATURIA caused by cyst rupture into renal pelvis, but can be caused by renal stone or UTI. - Recurrent bleeding may suggest the possibility of underlying renal cell carcinoma (especially age > 50) RENAL INFECTION suspected in Pt with flank pain, fever, and leukocytosis. Blood culture may be (+) while urinalysis is (-) because the cyst does not contact the urinary tract directly. NEPHROLITHIASIS HTN (50% of Pts have at time of presentation) cyst induced ischemia causes activation of the rennin-angiotensin system, and cyst decompression can lower BP temporarily. HTN and cardiac complication controlled with antihypertensives 10-15% of Pts have arterial CEREBRAL ANEURYSMS in the circle of Willis. Screening Arteriography not recommended unless Pt has a FHx or is undergoing elective surgery Other problems mitral valve prolapsed in up to 25% of Pts, aortic aneurysms, aortic valve abnormalities, colonic diverticula Medullary Sponge Kidney **dont confuse this with Medullary Cystic disease which is rare and autosomal recessive Caused by autosomal dominant mutation in the MCKD1 or MCKD2 genes on chromosomes 1 and 16 respectively th th Relatively common and benign d/o that is not usually Dx until the 4 or 5 decade, usually found incidentally on exam or X-ray It is a result of cystic dilation in the collecting tubules Clinical Presentation Kidneys have a marked irregular enlargement of the medullary and interpapillary collecting ducts - This is associated with diffuse medullary cysts that give a Swiss cheese appearance Present with gross or microscopic hematuria, recurrent UTIs or kidney stones Common abnormalities are a decreased urinary concentrating ability and nephrocalcinosis Incomplete type I distal renal tubular acidosis is a less common occurrence Colicky renal pain (generally from stones) Hematuria Resistant UTIs Lab findings Labs are usually normal, with occasional bacteriuria, hematuria, and crystals. Renal functions are NORMAL X-rays demonstrate lucencies on KUB and filling defects on IVP. Ultrasound demonstrates the small cysts well. CT scan is excellent at demonstrating the cyst. Imaging Dx is confirmed with IVP this shows striations in the papillary portions of the kidney produced by the accumulation of contrast in the dilated colleting ducts Management

There is no known therapy Adequate fluid intake (2L/d) helps prevent stone formation If hypercalciuria is present thiazide diuretics are recommended because they decrease calcium excretion Alkali therapy is recommended is renal tubular acidosis is present Management needed for the complicating factors of increased stone formation and resistant UTIs Underlying disease is usually benign, only a small %, <2%, develop serious complication where dialysis or transplant is required. Complications Renal function is usually well maintained unless there are complications from recurrent UTIs of nephrolithiasis. Solitary Cysts Simple cysts account for 65-70% of all renal masses Generally found at the outer cortex and contain fluid that is consistent with an ultrafiltrate of plasma Most cysts are incidentally found on ultrasonographic examination. Clinical Presentation S & S occur when the cyst obstruct urine flow, becomes infected or ruptures. Otherwise, most found incidentally. Common Sx may include pain in the back or flank that is dull and intermittent, non-tender abdominal mass (large cyst), if infected then pain in the flank, malaise and fever. Lab findings Usually normal. U/A may show microscopic hematuria. BUN and Creatinine are normal X-rays may demonstrate as incidental finding or as a filling defect on IVP CT scans are excellent at demonstrating the cyst. Ultrasound is also useful if the cyst is large enough Imaging Ultrasound and CT scanning are the recommended procedures for evaluating these masses Need to meet 3 criteria to be considered benign 1) Echo free 2) Sharply demarcated mass with smooth walls 3) An enhanced back wall (indicates good transmission through the cysts) Complex cysts have thick walls, calcifications, solid components, and mixed echogenicity. A simple cyst should have smooth, thin walls with no enhancement while a renal cell carcinoma will enhance, but is of lower density than the rest of the parenchyma. RCC are usually hypervascularized Arteriography can also be used to evaluate a mass preoperatively Management Main concern is differentiating them from malignancy, abscess, or polycystic kidney disease If cyst is found to be benign, periodic reevaluation is the standard of care If lesion not consistent with a simple cyst surgical exploration is recommended. Monitor and progress the cyst at regular intervals, episodic evaluation of urine or BUN/Creatinine and aggressive Tx of UTIs Usually benign Complications These simple cysts are generally asymptomatic but can become infected Cysts are problematic when infected or obstruct the ureter. They can develop in dialysis Pts. Can progress to malignancy Objective #9 State the etiology, clinical presentation, laboratory and imaging findings, management (including when to refer) and complications of: a. Chronic Kidney Disease (Aka Chronic Renal Failure CRF) pg. 803 b. Acute Renal Failure ARF (Chronic see Chronic Kidney Disease) pg. 797 Chronic Kidney Disease - CKD or Chronic Renal Failure - CRF (includes Uremia) Essentials of Diagnosis: *Progressive azotemia over months to years *Symptoms and signs of uremia when nearing end-stage disease *Hypertension in the majority *Isosthenuria and broad casts in urinary sediment are common *Bilateral small kidneys on ultrasound are diagnostic

Etiology

Clinical Presentation

Laboratory & Imaging Findings

Affects up to 20 million Americans OR 1 in 9 adults Most are unaware of the condition because they remain asx until disease has significantly progressed >70% of cases of late stage (stage 5) are due to DM or HTN Glomerulonephritis, cystic diseases and other urologic diseases account for 12% 15% of pts have other/unknown causes Rarely reversible and leads to progressive decline in renal fxn Reduction in renal mass lead to hypertrophy of remaining nephrons w/ hyperfiltration GFR transiently at supranormal levels in hypertrophied nephrons Burden placed on remaining nephrons progressive glomerular sclerosis/interstitial fibrosis Hyperfiltration may worsen renal fxn BUTdecreased renal mass in kidney donors NOT assoc. w/ chronic kidney disease Sxs often develop slowly and are nonspecific (see Table 22-7 Below) Individuals can remain asx until renal failure is far advanced (GFR < 10-15 mL/min) Manifestations: - Fatigue - Weakness - Malaise GI complaints (common): - Anorexia - N/V - Metallic taste in mouth - Hiccups Neuro problems: - Irritability - Difficulty concentrating - Insomnia - Subtle memory defects - Restless legs - Twitching Pruritus is common and difficult to tx As uremia progresses you see: - Decreased libido - Menstrual irregularities - Chest pain from pericarditis - Paresthesias Sxs of drug toxicity esp. for drugs renally excreted inc. as renal clearance worsens Physical exam - Pt appears chronically ill - Yellow skin and easy bruisability - HTN common - Uremic fetor characteristic fishy odor of the breath - Cardiopulmonary signs rales, cardiomegaly, edema, & pericardial friction rub - Mental status varies from decreased concentration to confusion, stupor, & coma - Myoclonus and Asterixis are additional signs of uremic fx on CNS Uremia used for this clinical syndrome but exact cause remains unknown BUN & SCr considered markers for unknown toxins In any pt it is important to identify and correct all reversible causes UTIs, obstruction, ECF volume depletion, Nephrotoxins, HTN, and CHF should be excluded (can worsen underlying chronic renal failure Labs: Dx of renal failure made by documenting elevations of BUN & SCr concentrations Evidence of previously elevated BUN & Cr, abnormal prior urinalyses, and stable but abnormal SCr on successive days is most consistent w/ chronic process Anemia, metabolic acidosis, Hyperphosphatemia, hypocalcemia, and hyperkalemia can occur with both acute/chronic renal failure U/A shows Isosthenuria

Management & When to Refer

Complications

Urinary sediment can show broad waxy casts as a result of dilated, hypertrophic nephrons Imaging: Small echogenic kidneys bilaterally (< 10cm) by U/S supports dx Protein restriction - Slows progression of ESRD - Must be careful of Cachexia Salt and water restriction - Kidney unable to adapt to large changes in Na intake - Daily intake of 1-2L of fluid maintains water balance Potassium restriction - Needed once GFR has fallen below 10-20mL/min Phosphorus restriction - Should be kept below 4.6mg/dL predialysis and 5.5 mg/dL when on dialysis - Limits foods rich in phosphorus (colas, eggs, dairy products, & meat) Magnesium restriction - Excreted mainly by kidneys - All Mg-containing laxatives/antacids are contraindicated in renal failure Dialysis Kidney Transplant Hyperkalemia Acid-Base Disorders Cardiovascular Complications - Hypertension - Pericarditis - CHF Hematologic Complications - Anemia - Coagulopathy Neurologic Complications Disorders of Mineral Metabolism Endocrine Disorders

Table 22-7: Symptoms and signs of uremia (she said to know this) Organ System Symptoms General Fatigue, weakness Skin Pruritis, easy bruisability ENT Eye Pulmonary Cardiovascular Metallic taste in mouth, epistaxis

Shortness of breath Dyspnea on exertion, retrosternal pain on inspiration (pericarditis) Gastrointestinal Anorexia, nausea, vomiting, hiccups Genitourinary Nocturia, impotence Isosthenuria* Neuromuscular Restless legs, numbness and cramps in legs Neurologic Generalized irritability to concentrate, Stupor, Asterixis, myoclonus, decreased libido peripheral neuropathy *Isosthenuria = a fixed specific gravity because the kidney is unable to dilute or concentrate the urine

Signs Sallow-appearing, chronically ill Pallor, ecchymoses, excoriations, xerosis Urinous breath Pale conjunctiva Rales, pleural effusion Hypertension, cardiomegaly, friction rub

Acute Renal Failure (ARF) aka Acute Kidney Injury Essentials of Diagnosis:

*Sudden increase in BUN or serum creatinine *Oliguria often associated *Symptoms and signs depend on cause Etiology Defined as a sudden decrease in renal fxn, resulting in an inability to maintain fluid and electrolyte balance and to excrete nitrogenous wastes SCr is a convenient marker - SCr will inc. by 1-1.5 mg/dL daily in absence of fxning kidneys - Can inc more rapidly in conditions like Rhabdomyolysis (up to 9x) Can be divided into three categories: - Prerenal azotemia o Most common cause of ARF (40-80% of cases) o Due to renal hypoperfusion o Appropriate physiologic change o If immediately reversed w/ renal blood flow restored renal parenchymal damage does NOT occur o If hypoperfusion persists ischemia intrinsic renal failure o Decreased renal perfusion occurs in one of three ways: Dec. in intravascular volume Causes of volume depletion include: o Hemorrhage o GI losses o Dehydration o Excessive diuresis o Extravascular space sequestration o Pancreatitis o Burns o Trauma o Peritonitis Change in vascular resistance Occur systematically with: o Sepsis o Anaphylaxis o Anesthesia o Afterload-reducing drugs (ACEIs, NSAIDs, EPI, NorEPI, high-dose dopamine, cyclosporine) Renal artery stenosis causes increased resistance and decreased perfusion Low cardiac output State of low effective renal arterial blood flow Occurs in states of: o Cardiogenic shock o CHF o PE o Pericardial tamponade o Arrhythmias & valvular disorders o Positive pressure ventilation (ICU setting) - Intrinsic renal disease o Account for 50% of all cases of ARF o Aka parenchymal dysfxn o Considered after prerenal & postrenal causes excluded o Sites of injury include: Tubules Insterstitium Vasculature Glomeruli - Postrenal azotemia o Least common cause of ARF (5-10% of cases) o Important to detect because of its reversibility o Occurs when urinary flow from both kidneys, or single fxning kidney, is obstructed

Clinical Presentation

Laboratory & Imaging Findings

Each nephron has elevated intraluminal pressure dec. GFR Causes include: Urethral obstruction Bladder dysfxn/obstruction Obstruction of both ureters/renal pelvises In men, benign prostatic hyperplasia is most common cause Pts taking anticholinergic are at higher risk Bladder, prostate, & cervical cancers Retroperitoneal processes Neurogenic bladder o Less common causes blood clots, bilateral ureteral stones, urethral stones/stricture, bilateral papillary necrosis Uremic milieu of ARF can cause specific sx When present, sx often due to azotemia or underlying cause Azotemia can cause: - N/V - Malaise - Altered sensorium HTN rare, but fluid homeostasis often altered Hypovolemia can cause prerenal disease Hypervolemia results from intrinsic/postrenal disease - May cause rales during lung exam Pericardial effusions/friction rub may be present with azotemia may result in cardiac tamponade Arrhythmias occur (hyperkalemia) Nonspecific diffuse abdominal pain/ileus and platelet dysfxn may be present bleeding more common in these pts Neuro exam - Encephalopathic changes - Asterixis - Confusion - Seizures Postrenal Azotemia - Pts may be anuric or polyuric - May complain of lower abdominal pain - Obstruction can be partial, intermittent, or complete - Pt may have enlarged prostate, distended bladder, or mass detected on pelvic exam Elevated BUN & creatinine present (does not distinguish ARF from CRF) Hyperkalemia occurs from impaired renal K+ excretion ECG - Peaked T waves - PR prolongation - QRS widening - Long QT segment seen with hypocalcemia Anion gap metabolic acidosis Hyperphosphatemia occurs when phosphorus cant be secreted by damaged tubules w/ or w/o cell catabolism Hypocalcemia w/ metastatic Ca phosphate deposition may be observed when product of Ca and phosphorus > 70mg/dL Anemia occurs as result of dec. EPO production over wks assoc. platelet dysfxn Prerenal Azotemia - BUN:creatinine ratio will exceed 20:1 bc of inc. urea reabsorption - With dec. GFR the kidney will reabsorb water/salt avidly if no intrinsic tubular dysfxn Postrenal Azotemia - May initially reveal high urine osmolality, low urine Na, & high BUN:creatinine ratio - Urine Na will inc. after several days as kidneys fail and cannot concentrate urine isosthenura - Urine sediment usually benign - Pts should undergo bladder ultrasonography and bladder cath of hydroureter/hydronephrosis present with enlarged bladder o o

Management & When to Refer

Identifying cause is 1 step towards tx the pt Prerenal Azotemia - Depends on cause but always: o Maintenance of euvolemia o Pay attn to serum K+ o Avoid nephrotoxic drugs - Careful assessment of volume status, drug usage, and cardiac fxn Postrenal Azotemia - Pts usually undergo postobstructive diuresis care should be take to avoid dehydration - Prompt tx of obstruction w/in days by cath, stents, or other surgical procedures can result in complete reversal of acute process
st

Table 22-4: Classification & Differential Diagnosis of ARF Prerenal Postrenal Azotemia Azotemia Etiology Poor renal perfusion >20:1 >500 Obstruction of urinary tract >20:1 <400

Acute Tubular Necrosis Ischemia, Nephrotoxins <20:1 250-300

Serum BUN:Cr ratio Urine Osmolality (mosm/kg) Urinary sediment

Intrinsic Renal Disease Acute Acute Interstitial Glomerulonephritis Nephritis Poststreptococcal; Allergic rxn; drug rxn collagen-vascular disease >20:1 <20:1 Variable Variable

Benign or hyaline casts

Normal or red cells, white cells, or crystals

Granular (muddy brown) casts, renal tubular casts

Dysmorphic red cells and red cell casts

White cells, white cell casts, with or w/o eosinophils

Objective #10 Recognize etiology, signs and symptoms, appropriate lab findings, and complications of: a. Metabolic Acidosis and Alkalosis (pgs. 785 792) b. Hyper and hyponatremia (including volume overload) (pgs. 767 772) c. Hyper and hypokalemia (pgs. 773-776) d. Hyper and hypocalcemia (pgs. 777-780) e. Hyper and hypomagnesemia (pgs. 793-784) f. Calculate Acid-Base problems given relevant lab findings (Dr. Lowes Lecture) Metabolic Acidosis MA (See table 21-13 pg. 786) Essentials of Diagnosis: *Decreased HCO3- with acidemia *Classified into high anion gap acidosis and normal anion gap acidosis *The highest anion gap acidoses are seen in lactic acidosis, Ketoacidosis, or toxins *Normal anion gap acidosis is mainly caused by gastrointestinal HCO3- loss or RTA. Urinary anion gap may help distinguish between these causes Etiology Hallmark of MA is decreased HCO3 Calculation of anion gap helps determine cause Anion gap represents difference b/w readily measured anions and cations Anion gap = Na+ - (HCO3 + Cl ) normally between 12 4 mEq/L - Unmeasured cations are Ca, Mg, gamma-globulins, and K - Unmeasured anions are negatively charged albumin, phosphate, sulfate, lactate, & other organic anions - Can be misleading - Non-acid-base disorders may contribute to error (hypoalbuminemia, hyperNa, or hypoNa) Increased Anion Gap Acidosis (Increased Unmeasured Anions) Hallmark of this disorder is that MA is associated with normal serum Cl-, so the anion gap increases

Normochloremic MA usually results from addition to blood of nonchloride acids like - Lactate - Acetoacetate - Beta-hydroxybutyrate - Exogenous toxins Lactic Acidosis - Lactic acid formed from pyruvate in anaerobic glycolysis - Most lactate produced in tissues with high rates of glycolysis (gut, skeletal muscle, brain, skin, and RBCs) - Normal Function o Lactate levels stay low (1 mEq/L) o Metabolism by liver thru gluconeogenesis or oxidation o Kidneys metabolize ~30% - Lactic Acidosis o Lactate levels are at least 4-5 mEq/L but commonly up to 10-30 mEq/L o Mortality exceeds 50% o Two types of lactic acidosis (both assoc. w/ inc. lactate production & dec. lactate utilization): Type A (hypoxic) More common type Results from o Poor tissue perfusion o Cardiogenic, septic, or hemorrhagic shock o Carbon Monoxide or cyanide poisoning Cause lactic acid production to inc. peripherally AND hepatic metabolism of lactate to decrease as liver perfusion declines Severe form impairs liver to extract perfused lactate Type B May be due to metabolic causes o DM o Ketoacidosis o Liver disease o Renal failure o Infection o Leukemia o Lymphoma o Toxicity (ethanol, methanol, salicylates, metformin) Nutritional problems are an important cause (parenteral nutrition w/o thiamine is cause bc of deranged metabolism of pyruvate) Seen in AIDS pts Idiopathic seen in debilitated pts and has extremely high mortality rate Diabetic Ketoacidosis - Characterized by hyperglycemia and MA (pH <7.25 or plasma bicarb < 16 mEq/L) - Anion gap MA is acid-base disturbance usually ascribed to diabetic Ketoacidosis + H +B +NaHCO3 CO2 + NaB +H2O - B is beta-hydroxybutyrate or acetoacetate - Anion gap should be calculated from serum electrolytes as measured bc correction of serum Na for dilutional fx of hyperglycemia will incorrectly exaggerate anion gap - Increased anion gap is due to hyperketonemia and inc. in serum lactate 2 to reduced tissue perfusion/inc. anaerobic metabolism - During recovery phase, anion gap acidosis can be transformed into hyperchloremic non-anion gap acidosis o GFR inc. from NaCl tx o Retention of Cl causes mild dec. in anion gap from dilution o Inc. GFR causes urinary excretion of ketone salts (NaB) (formed as bicarb consumed) - Kidney resorbs ketone anions poorly but can compensate for loss of anions by inc. resorption of Cl - Pts may have normal renal perfusion but marked ketonuria, severe MA, and only mildly inc. anion gap - Pts clinical status and reduction of anion gap are better markers of improvement than monitoring serum acetone test Alcoholic Ketoacidosis

Common d/o of chronically malnourished pts who consume large quantities of alcohol daily Usually have mixed acid-base d/o Decreased bicarb is usual 50% of pts have normal or alkalemic pH Three types of MA seen in alcoholic ketoacidosis: o Ketoacidosis due to beta-hydroxybutyrate and acetoacetate excess o Lactic acidosis inc. NADH:NAD ratio causes inc. production/dec. utilization of lactate o Hyperchloremic acidosis from bicarb loss in urine assoc. with ketonuria - of the pts have either hyperglycemia or hypoglycemia - When serum glucose levels are >250 mg/dL, the distinction from diabetic ketoacidosis is hard - Dx of alcoholic ketoacidosis supported by absence of DM hx and by no evidence of glucose intolerance after initial therapy Toxins - Multiple toxins/drugs can inc. anion gap by inc. endogenous acid production (ethanol, methanol, salicylates, metformin) Uremic Acidosis - At GFR < 20mL/min, the inability to excrete H+ w/ retention of acid anions results in inc. anion gap acidosis (rarely severe) - Unmeasured anions replace bicarb - Hyperchloremic normal anion gap acidosis develops in earlier stages of CKD Normal Anion Gap Acidosis Hallmark of this d/o is that the low bicarb of MA is assoc. with hyperchloremia, so anion gap remains normal Most common causes GI bicarb loss and defects in renal acidification (renal tubular acidosis RTA) - Urinary anion gap helps differentiate b/w the two GI HCO3 Loss - Bicarb is excreted in multiple areas in GI tract (small bowel/pancreatic secretions contain large amts) - Massive diarrhea or pancreatic drainage can result in bicarb loss bc of inc. excretion/dec. absorption - Hyperchloremia occurs bc ileum/colon secrete bicarb in 1:1 exchange for Cl (countertransport) - Resultant vol. contraction causes further inc. Cl retention by kidney in setting of dec. anion bicarb Renal Tubular Acidosis (RTA) - Hyperchloremic acidosis w/ normal anion gap and normal GFR, and in absence of diarrhea defines RTA - Defects o Inability to excrete H+ o Inappropriate reabsorption of bicarb - Three major types differentiated by clinical setting, urinary pH, urinary anion gap, and serum K+ level (type III RTA not used anymore): o Classic distal RTA (type I) characterized by hypoK hyperCl MA due to selective deficiency in H+ secretion in alpha intercalated cells in the collecting tubule o Proximal RTA (type II) hypoK hyperCl MA due to selective defect in proximal tubules ability to adequately reabsorb filtered bicarb (HCO3 ) o Hyporeninemic hypoaldosteronemic RTA (type IV) Most common form of RTA in clinical practice Only type characterized by hyperK, hyperCl acidosis Defect is aldosterone deficiency/antagonism impairs distal nephron Na+ reabsorption and K+/H+ excretion Dilutional Acidosis rapid dilution of plasma vol by 0.9% NaCl mild hyperCl acidosis Assessment of Hyperchloremic Metabolic Acidosis by Urinary Anion Gap + Inc. renal NH4 Cl excretion to enhance H+ removal is a normal physiologic response to MA Normal daily urinary excretion of NH4Cl of about 30 mEq can be inc. up to 200 mEq in response to acid load Urinary anion gap from random urine sample reflects ability of kidney to excrete NH 4Cl - Helps distinguish b/w renal and GI causes of hyperchloremic acidosis - If cause of MA is GI bicarb loss (diarrhea) the renal acidification ability remains normal and NH4Cl excretion inc. in response to acidosis (urinary anion gap is negative) - If cause is distal RTA, urinary anion gap is positive bc the basic lesion in d/o is the inability of kidney to excrete H+ and so inability to inc. NH4Cl excretion

Signs & Symptoms Lab Findings

Complications

If cause is proximal RTA the urinary anion gap is negative bc kidney has defective bicarb reabsorption, leading to inc. bicarb excretion When vol. depletion is present, the urinary anion gap is a better measurement of ability to acidify urine than urinary pH (if large amts of other anions present in urine urinary anion gap may not be reliable) Sx of MA are mainly those of the underlying disorder Compensatory hyperventilation is an important clinical sign - May be misinterpreted as primary respiratory disorder - When severe, Kussmaul respirations seen (deep, regular, sighing respirations) Blood pH, serum HCO3-, and PCO2 are decreased Anion gap may be normal (Hyperchloremic) or increased (normochloremic) HyperK may be seen

Metabolic Alkalosis (See table 21-15, pg. 791) Essentials of Diagnosis: *High HCO3 with alkalemia *Evaluate effective circulating volume by physical examination and check urinary chloride concentration. This will help differentiate saline-responsive metabolic alkalosis from saline-unresponsive alkalosis Etiology Characterized by high bicarb Initiation factors - abnormalities that generate bicarb w/in the body Maintenance factors abnormalities that promote renal conservation of bicarb Metabolic alkalosis may remain even after initiation factors disappear Classified into two groups based saline responsiveness (urinary Cl-) which are markers for volume status Saline-Responsive Metabolic Alkalosis (extracellular volume contraction) - More common disorder - Characterized by normotensive extracellular vol. contraction and hypokalemia (hypotension or orthostatic hypotension may be seen) - Vomiting/nasogastric suction loss of acid (HCl) causes alkalosis o Vol contraction from loss of Cl- sustains alkalosis bc decline in GFR cause avid renal Na+ and bicarb reabsorption o Cl- depletion from stomach so available anion is bicarb which has inc. reabsorption proximally and urine pH may be acidic (paradoxic aciduria) - Urinary Cl- is preferred to urinary Na+ as a measure of extracellular volume - Generally assoc. w/ hypokalemia (direct fx on renal excretion of K+ and hyperaldosteronism from vol. depletion) o HypoK worsens metabolic alkalosis by inc. bicarb reabsorption in proximal tubule and H+ ion secretion in distal tubule o Repletion of KCl reverses d/o Saline-Unresponsive Alkalosis (implies a volume expanded state) - Hyperaldosteronism o 1 hyperaldosteronism causes expansion of extracellular vol. with HTN o Metabolic alkalosis with hypoK results from renal mineralocorticoid fx o High levels of NaCl excreted (urinary Cl- is high >20 mEq/L) bodies attempt to dec. extracellular vol. - Alkali administration with decreased GFR o Enhanced bicarbonaturia prevents pts w/ normal renal fxn from developing metabolic alkalosis (even with large bicarb ingestion) o Pts with CKD have inadequate urinary excretion of bicarb (if antacids are consumed metabolic alkalosis will occur) o Volume contraction from renal hyperCa fx further exacerbate alkalosis Signs No characteristic signs/sx & Orthostatic hypotension may be seen + Symptoms Weakness and hyporeflexia occur is serum K is markedly low Tetany and neuromuscular irritability (rare)

Lab Findings

Complications

Arterial blood pH & bicarb are elevated Arterial PCO2 increased + Serum K and Cl decreased May be increased anion gap

Hypernatremia Essential of Diagnosis: *Increased thirst and water intake is the first defense against hypernatremia *Urine osmolality helps differentiate renal from nonrenal water loss Etiology Defined as sodium concentration > 145 mEq/L All pts have hyperosmolality Pt is typically hypovolemic due to free water losses (but can see hypervolemic hypernatremia) Excessive Na intake is a rare cause Mild in primary aldosteronism (usually no sx) Intact thirst mechanism and access to water are primary defense Hypothalamus senses minimal changes in serum osmolality, triggering thirst mechanism and inc. water intake Underlying disorders include dehydration, lactulose/Mannitol therapy, central/nephrogenic diabetes insipidus Excess water loss can cause hypernatremia only when adequate water intake not possible unconscious pts Signs Dehydrated pt orthostatic hypotension/oliguria are common findings & Sx may be delayed since water shifts from cells to intravascular space to protect volume status Symptoms Early signs: - Lethargy - Irritability - weakness Sx seen with severe hypernatremia (Na > 158 mEq/L): - Hyperthermia - Delirium - Seizures - Coma Sx in elderly nonspecific - Recent change in consciousness poor prognosis Lab Findings Urine osmolality > 400 mosm/kg - renal water conserving ability is fxning - Nonrenal losses hyperNa occurs if water intake falls behind hypotonic fluid losses from excessive sweating, respiratory tract, or bowel movt & lactulose causes osmotic diarrhea with loss of free water - Renal losses severe hyperglycemia is cause and progressive vol. depletion from osmotic glucosuria, osmotic diuresis occurs with use of Mannitol or urea Urine osmolality < 250 mosm/kg - Dilute urine characteristic of central/nephrogenic DI - Result from renal insensitivity to ADH - Common causes lithium, Demeclocycline, relief of urinary obstruction, interstitial nephritis, hypercalcemia, and hypokalemia Complications Osmotic demyelination is an uncommon consequence of severe hypernatremia

Volume Overload (pg. 772) Essentials of Diagnosis: *Disorder of excessive sodium retention in the setting of low arterial underfilling (e.g., CHF or cirrhosis) *Hyponatremia from water retention in edematous states is associated with sodium retention Hallmark of volume overloaded state is sodium retention Abnormally low arterial filling (CHF or cirrhosis) activates neurohumeral axis Renin-angiotensin-aldosterone system, SNS, & ADH release are stimulated Na retention with edema results ADH release stimulus is nonosmotic ADH released in response to baroreceptors activation (dec. in arterial baroreceptors stretch)

ADH stimulates renal V2 receptors inc. water reabsorption Hyponatremia can develop in edematous state

Hyponatremia Essentials of Diagnosis: *The patients volume status and serum osmolality are essential to determine etiology *Hyponatremia usually reflects excess water retention relative to sodium rather than sodium deficiency *Hypotonic fluids commonly cause hyponatremia in hospitalized patients *Treatment strategy should be based on pathophysiology, symptoms, severity, and acuity Etiology Defined as a serum sodium concentration < 135 mEq/L Most common electrolyte abnormality in hospitalized pts Mismanagement can result in catastrophic neurologic consequences due to cerebral osmotic demyelination Etiology of most cases apparent thru H&P, and basic labs Most cases are hypotonic Total body water and total body Na can be low, normal, or high in hyponatremia since the kidney can regulate Na and water homeostasis independently Most cases reflect water imbalance/abnormal water handling, NOT Na imbalance Eval starts w/ careful hx: - New meds - Changes in fluid intake (polydipsia, anorexia, IV fluid rates and composition) - Fluid output (N/V/D, ostomy output, polyuria, oliguria, insensible losses) Physical exam categorize pts vol. status into hypovolemia, euvolemia, or hypervolemia Hypovolemic Hypotonic Hyponatremia Occurs with renal or extrarenal volume loss and hypotonic fluid replacement Total body Na and total body water are decreased ADH secretion inc. to maintain intravascular volume causes free water retention from hypotonic fluid replacement Body sacrifices serum osmolality to preserve intravascular volume Losses of water/Na are replaced by water alone Cerebral salt wasting is rare subset of disease Euvolemic Hypotonic Hyponatremia Broad differential dx (only need to focus on hormone abnormalities hypothyroid) Hormone abnormalities - Hypothyroidism/adrenal insufficiency can cause hyponatremia (may be related to ADH) Sx depend on severity/acuity Chronic disease can be severe (Na concentration < 110 mEq/L) but may be asx bc brain adapts by decreasing tonicity over wks/mos Acute disease develops over hours to days and can be severely sx w/ relatively moderate hyponatremia Mild cases (Na concentrations of 135-135 mEq/L) is usually asx Mild sx of nausea/malaise progress to: - Headache - Lethargy - Disorientation Most serious sx include: - Respiratory arrest - Seizure - Coma - Permanent brain damage - Brain stem herniation - Death Premenopausal women more likely than menopausal women to die/suffer permanent brain injury from hyponatremic encephalopathy Serum electrolytes SCr

Signs & Symptoms

Lab Findings

Complications

Serum osmolality Urine sodium May need to do thyroid/adrenal fxn tests Most serious complication is iatrogenic cerebral osmotic demyelination from overly rapid/inappropriate Na correction

Hyperkalemia Essentials of Diagnosis: *Hyperkalemia may develop in patients taking ACE inhibitors, ARBs, potassium sparing diuretics, or their combination, even with no or only mild renal dysfunction *The ECG may show peaked T waves, widened QRS and biphasic QRS-T complexes, or may be normal despite life-threatening hyperkalemia *Measurement of plasma potassium level differentiates potassium leak from blood cells in cases of clotting, leukocytosis, and thrombocytosis from elevated serum potassium *Rule out extracellular potassium shift from the cells in acidosis and assess renal potassium excretion Etiology Usually develops in pts with advanced renal dysfxn but can also develop w/ no/mild renal dysfxn Intracellular K+ shifts to ECF in hyperK associated with acidosis - Serum K+ concentration rises ~0.7 mEq/L for every decrease of 0.1 pH unit Repeatedly clenching/unclenching fist during venipuncture may raise K+ concentration by 1-2 mEq/L by causing acidosis and K+ loss from cells Absence of acidosis serum K+ concentration rises ~1 mEq/L when total body K+ excess of 1-4 mEq/kg The higher the serum K+ concentration, the smaller the excess necessary to raise K+ levels further Mineralocorticoid deficiency in Addison disease or CKD is another cause w/ decreased renal excretion of K+ ACEI/ARBs used for pts with CHF or CKD is a cause Simultaneous use of Spironolactone, Beta-blockers further increases risk Thiazide/loop diuretics, Na bicarb may help minimize hyperK Milder hyperK that recurs in pts NOT on ACEIs is usually due type IV renal tubular acidosis (RTA) Heparin is another cause (inhibits aldosterone production) TMP inhibits renal excretion of K+ causing hyperK but K+ concentration returns to baseline when stop drug Immunosuppressive drugs can cause hyperK in organ transplant pts (esp. kidney transplant) Severe hyperK and cardiovascular disturbances caused by use of drugs with K ATP channel opening properties (Kchannel syndrome) Commonly seen in HIV pts Signs Elevated K+ concentration interferes with normal neuromuscular fxn producing & - Muscle weakness Symptoms - Flaccid paralysis (rare) - Abdominal distention - Diarrhea Lab Findings ECG is not a sensitive method for detecting hyperK bc of pts w/ serum concentration > 6.5 mEq/L will not have any ECG changes ECG changes - Peaked T waves of increased amplitude - Widening of QRS and biphasic QRS-T complexes - Inhibition of atrial depolarization may occur - Heart rate may be slow Complications VFib and cardiac arrest (terminal events)

Hypokalemia Essentials of Diagnosis: *Severe hypokalemia may induce dangerous arrhythmias and Rhabdomyolysis *Transtubular potassium concentration gradient (TTKG) can distinguish renal from nonrenal loss of potassium Etiology Most common cause (esp. in developing countries) is GI loss due to infectious diarrhea K+ concentration in intestinal secretion is 10x higher (80mEq/L) than gastric juice Aldosterone is most important regulator of body K+ content

Signs & Symptoms

Lab Findings

Complications

Can occur as result of: Shifting of K+ intracellularly from extracellular space - Extrarenal K+ loss (insufficient K+ intake) - Renal K+ loss K+ uptake by cell stimulated by insulin in presence of glucose Facilitated by beta-adrenergic stimulation (alpha-adrenergic stimulation blocks K+ uptake) Self-ltd occurs in 50-60% of all trauma pts (enhanced release of EPI) Profound hypoK due to barium or cesium intoxication may be result of transport of K+ into cells In presence of acidosis suggests profound K+ depletion (tx ASAP) Increases likelihood of digitalis toxicity Some genetic mutations that affect fluid/electrolyte metabolism reported as cause Magnesium is important cofactor for K+ uptake & maintenance of intracellular K+ levels Loop diuretics cause inc. K+/Mg losses Mg depletions suspected in persistent hypoK refractory to K+ repletion Mild moderate hypokalemia sx: - Muscular weakness - Fatigue - Muscle cramps - Smooth muscle involvement constipation or ileus Severe hypoK sx (< 2.5 mEq/L): - Flaccid paralysis - Hyporeflexia - Hypercapnia - Tetany - Rhabdomyolysis HTN helps dx hypoK caused by aldosterone/mineralcorticoid excess Urinary K+ concentration is low (< 20 mEq/L) as result of extrarenal fluid loss (V/D) Urinary K+ concentration is high (>40mEq/L) with urinary losses (mineralocorticoid excess, Bartter syndrome, Liddle syndrome) TTKG > 4 suggest renal K+ loss with inc. distal K+ secretion ECG findings: - Dec. amplitude - Broadening of T waves - Prominent U waves - Premature ventricular contractions - Depressed ST segments Hypomagnesemia Arrhythmias

Hypercalcemia Essentials of Diagnosis: *Primary hyperparathyroidism and malignancy-associated hypercalcemia are the most common causes *hypercalciuria usually precedes hypercalcemia *Most often, asymptomatic, mild hypercalcemia ( 11 mg/dL) is due to primary hyperparathyroidism, whereas the symptomatic, severe hypercalcemia ( 14 mg/dL) is due to hypercalcemia of malignancy Etiology Important causes (Table 21-8) - Increased intake or absorption (Vitamin D or A excess) - Endocrine disorders (primary hyperparathyroidism, CKD, acromegaly, adrenal insufficiency) - Neoplastic diseases (PTHrP producing tumors) - Miscellaneous causes (complication of renal transplant) Primary hyperparathyroidism and malignancy account for 90% of all cases of hyperCa - Primary hyperparathyroidism most common cause in ambulatory pts Chronic hyperCa (>6mos) or nephrolithiasis benign cause PTHrP producing tumors is most common paraneoplastic endocrine syndrome most cases in inpatients - Prognosis poor

Signs & Symptoms

Lab Findings

Complications

HyperCa also causes nephrogenic DI and vol. depletion worsens hyperCa May affect GI, renal, and neuro fxn Focus of H&P should be on duration of process of hyperCa and evidence of neoplasm Mild hyperCa often asx Sx usually occur if serum Ca is >12 mg/dL and tend to be more severe if hyperCa develops acutely Sx always are constipation and polyuria Hypocalciuric hyperCa polyuria absent GI sx include: - N/V - Anorexia - Peptic ulcer disease Renal colic or hematuria from nephrolithiasis may be seen Volume depletion/azotemia nephrogenic DI Neuro manifestations include: - Mild drowsiness - Weakness - Depression - Lethargy - Stupor - Coma (severe hyperCa) Ventricular extrasystoles and idioventricular rhythm (accentuated by digitalis) ECG findings: - Shortened QT interval Significant elevation of serum Ca seen - Level must be interpreted in relation to serum albumin level Highest serum Ca levels (>15 mg/dL) usually seen in malignancy > 200 mg/d of urinary Ca excretion suggest hypercalciuria & <100 mg/d is hypocalciuria Serum phosphate may/may not be low (depends on cause) Measurement of PTH and PTHrP distingquish b/w causes - Suppressed PTH/elevated PTHrP = malignancy-assoc. hyperCa - Elevated PTH = hyperparathyroidism ECG changes?

Hypocalcemia Essentials of Diagnosis: *Often mistaken as a neurological disorder *Check for decreased PTH, vitamin D, or magnesium depletion *If the ionized calcium level is normal despite a low total serum calcium, calcium metabolism is usually normal Etiology Development of true hypoCa (decreased ionized Ca) implies insufficient action of PTH or active vitamin D Most common cause of low total serum Ca concentration is hypoalbuminemia Correction of serum Ca concentration is need to accurately reflect ionized Ca concentration 2+ 2+ When serum albumin concentration is < 4 g/dL, serum Ca concentration is depressed in a ration of 0.8-1 mg of Ca to 1 g of albumin Important causes (Table 21-7) - Decreased intake or absorption (malabsorption or vitamin D deficient) - Increased loss (CKD, diuretic therapy) - Endocrine disease (hypoparathyroidism or familial hypocalcemia) - Physiologic cause (decreased serum albumin, decreased end-organ response to vitamin D, hyperphosphatemia) The most common cause is the advanced stages of CKD - Dec. production of active vitamin D3 and hyperphosphatemia both play a role 1 hypoparathyroidism - Mutation of Ca-sensing receptor causing inappropriate suppression of PTH release leads to hypoCa Magnesium depletion reduces PTH release/tissue responsiveness to PTH causing hypoCa HypoCa in pancreatitis severe disease Elderly hospitalized pts w/ low ionized serum Ca and hypophosphatemia, w/ or w/o elevated PTH are most likely

Signs & Symptoms

Lab Findings

Complications

vitamin D deficient Increases excitation of nerve and muscle cells mainly fx neuromuscular and cardiovascular systems Extensive spasm of skeletal muscle causes cramps/tetany Laryngospasm w/ stridor can obstruct airway Convulsion, paresthesias of lips/extremities, and abdominal pain can occur Chvostek sign - Contraction of facial muscle in response to tapping facial nerve anterior to ear Trousseau sign - Carpal spasm occurring after occlusion of brachial artery with BP cuff for 3min If chronic hypoparathyroidism cataracts and calcification of basal ganglia of brain may appear ECG changes - Prolongation of QT interval (due to lengthened ST segment) - Ventricular arrhythmias Serum Ca concentration is low (< 9mg/dL) In true hypoCa, ionized serum Ca concentration is also low (< 4.7 mg/dL) Serum phosphate is usually elevated in hypoparathyroidism or advanced stages of CKD (suppressed in early stages of CKD or vitamin D deficiency) Serum Mg concentration commonly low Respiratory alkalosis total serum Ca normal but ionized Ca low Ventricular arrhythmias

Hypermagnesemia Essentials of Diagnosis: *Almost always associated with advanced stages of CKD and a history of chronic intake of magnesium containing drugs Etiology Mg excess almost always result of advanced CKD and inability to excrete what has been taken in from food/drugs (esp. long-term use of antacids/laxatives) Signs Characteristic manifestations: & - Muscle weakness (may see flaccid paralysis) Symptoms - Decreased DTRs - Mental obtundation - Confusion - Ileus - Urinary retention - Hypotension Lab Findings Serum Mg2+ is elevated In common setting of CKD - SCr and BUN, phosphate, and uric acid are elevated (serum K+ may also be elevated) Serum Ca2+ low ECG changes (probably related to associated hyperK): - Increased PR interval - Broadened QRS complexes - Peaked T waves Complications Respiratory muscle paralysis or cardiac arrest

Hypomagnesemia Essentials of Diagnosis: *Serum concentration of magnesium may not be decreased even in the presence of magnesium depletion. Check urinary magnesium excretion if renal magnesium wasting is suspected *Causes neurologic symptoms and arrhythmias *Impairs release of PTH Etiology Causes (Table 21-11) - Diminished absorption or intake (malabsorption, chronic diarrhea) - Increased renal loss (hyperaldosteronism, hyperthyroidism, hyperparathyroidism, hypercalcemia, volume expansion, tubulointerstitial diseases)

Signs & Symptoms

Lab Findings

Complications

Normomagnesemia does not exclude Mg depletion bc only 1% of total body Mg is in ECF ~50% of all hospitalized pts w/ ordered electrolytes have unrecognized hypoMg ~40% of pts will also have hypoK & ~50% will have hypoCa Renal K wasting occurs due to hypoMg will not get better until hypoMg tx Suppresses PTH release and causes end-organ resistance to it and to low vitamin D3 levels Common sx are those of hypoK and hypoCa: - Weakness - Muscle cramps Marked neuromuscular and CNS hyperirritability - Tremors - Athetoid movements - Jerking - Nystagmus - (+) Babinski response HTN, tachycardia, and ventricular arrhythmias may be seen Confusion/disorientation prominent features Urinary excretion of Mg > 10-30 mg/d or fraction excretion >2% indicates renal Mg wasting HypoCa and hypoK present ECG changes: - Prolonged QT interval from lengthening of ST segment PTH secretion usually suppressed Arrhythmias

You might also like